Question

Calculate the net torque (magnitude and direction) on the beam in the figure below about the...

Calculate the net torque (magnitude and direction) on the beam in the figure below about the following axes. (Assume α = 43°, β = 28°, γ = 16°, ℓ = 3.6 m) (a) an axis through O perpendicular to the page (b) an axis through C perpendicular to the page

11-p-005-alt.gif

1 0
Add a comment Improve this question Transcribed image text
✔ Recommended Answer
Answer #1

(a)The torque about axis through O is, T =-30 sin a (0) + 25 cos B(1) - 10sin y ( ) = 0+(25)cos (28)(1.8)-(10) sin (16)(3.6)

Add a comment
Know the answer?
Add Answer to:
Calculate the net torque (magnitude and direction) on the beam in the figure below about the...
Your Answer:

Post as a guest

Your Name:

What's your source?

Earn Coins

Coins can be redeemed for fabulous gifts.

Similar Homework Help Questions
ADVERTISEMENT
Free Homework Help App
Download From Google Play
Scan Your Homework
to Get Instant Free Answers
Need Online Homework Help?
Ask a Question
Get Answers For Free
Most questions answered within 3 hours.
ADVERTISEMENT
ADVERTISEMENT
ADVERTISEMENT